Weyl Spinors Transformation, QFT1, Peskin, Chapter 3

In summary, the conversation is about proving the identity $\sigma^2 \vec\sigma^* = -\vec \sigma \sigma^2$, which is crucial for the transformation properties of $\psi_L$ and $\psi_R$. The main problem is that the identity is not immediately obvious and requires matrix multiplication to prove. The conversation also touches on the use of Grassmann-number valued fields for fermions and the significance of antisymmetric products in these calculations. Finally, there is a question about why we can consider the transformation of $\psi_L$ alone and then multiply by $\sigma^2$, rather than considering the transformation of the entire object $\sigma^2 \psi_L^*$.
  • #1
Pouramat
28
1
Homework Statement
The transformation law for Weyl spinors is as following (3.37); these transformation laws are connected by complex conjugation; using the identity (3.38)
Relevant Equations
(3.37) and (3.38) peskin
\begin{align}
\psi_L \rightarrow (1-i \vec{\theta} . \frac{{\vec\sigma}}{2} - \vec\beta . \frac{\vec\sigma}{2}) \psi_L \\
\psi_R \rightarrow (1-i \vec{\theta} . \frac{{\vec\sigma}}{2} + \vec\beta . \frac{\vec\sigma}{2}) \psi_R
\end{align}
I really cannot evaluate these from boost and rotation generator which was introduced in (3.26) and (3.27) Peskin.
Although the main porblem is the identity introduced below:
$$
\sigma^2 \vec\sigma^* = -\vec \sigma \sigma^2
$$
My attempt:
$$
\sigma^1 =
\begin{pmatrix}
0 & 1 \\
1 & 0 \\
\end{pmatrix};
\sigma^2 =
\begin{pmatrix}
0 & -i \\
i & 0 \\
\end{pmatrix};
\sigma^3 =
\begin{pmatrix}
1 & 0 \\
0 & 1 \\
\end{pmatrix}
$$
the h.c. of the Pauli Sigma matrices is as following:
$$
(\sigma^1)^* =
\begin{pmatrix}
0 & 1 \\
1 & 0 \\
\end{pmatrix};
(\sigma^2)^* =
\begin{pmatrix}
0 & i \\
-i & 0 \\
\end{pmatrix};
(\sigma^3)^* =
\begin{pmatrix}
1 & 0 \\
0 & 1 \\
\end{pmatrix}
$$
so ##\sigma^2 = diag(1,1)##,right?
I think my problem is that I cannot write the identity in components form.
 
Last edited:
Physics news on Phys.org
  • #2
Of course ##\sigma^3=\mathrm{diag}(1,-1)##. I'd say the most simple way is just to prove the equation by doing the matrix multiplications for the three matrices :-).
 
  • Like
Likes Pouramat
  • #3
vanhees71 said:
Of course ##\sigma^3=\mathrm{diag}(1,-1)##. I'd say the most simple way is just to prove the equation by doing the matrix multiplications for the three matrices :-).
Ohhh, Yes, it was a typo :)
the LHS:
$$
\large \begin{pmatrix}
0 & 1 \\
1 & 0 \\
\end{pmatrix}
+
\begin{pmatrix}
0 & i \\
-i & 0 \\
\end{pmatrix}
+
\begin{pmatrix}
1 & 0 \\
0 & -1 \\
\end{pmatrix} =
\begin{pmatrix}
1 & 1+i \\
1-i & -1 \\
\end{pmatrix}
$$
which should be equal to RHS:
$$
-\large( \begin{pmatrix}
0 & 1 \\
1 & 0 \\
\end{pmatrix}
+
\begin{pmatrix}
0 & -i \\
i & 0 \\
\end{pmatrix}
+
\begin{pmatrix}
1 & 0 \\
0 & -1 \\
\end{pmatrix})
=
\begin{pmatrix}
-1 & 1+i \\
1-i & 1 \\
\end{pmatrix}
$$
but it is not!
 
  • #4
I have no idea what you are doing here. You are supposed to show that (I'll give one example)
$$\sigma^2 \sigma^{1*}=-\sigma^1 \sigma^2.$$
Take the right-hand side and use the anti-commutation relations
$$\{\sigma^j,\sigma^k\}=2 \delta^{jk}$$
to get
$$-\sigma^1 \sigma^2 =+ \sigma^2 \sigma^1.$$
Since ##\sigma^1## is real this hows that
$$-\sigma^1 \sigma^2=\sigma^2 \sigma^{1*}.$$
Now you can prove the other two identities!
 
  • Informative
Likes Pouramat
  • #5
vanhees71 said:
I have no idea what you are doing here. You are supposed to show that (I'll give one example)
$$\sigma^2 \sigma^{1*}=-\sigma^1 \sigma^2.$$
Take the right-hand side and use the anti-commutation relations
$$\{\sigma^j,\sigma^k\}=2 \delta^{jk}$$
to get
$$-\sigma^1 \sigma^2 =+ \sigma^2 \sigma^1.$$
Since ##\sigma^1## is real this hows that
$$-\sigma^1 \sigma^2=\sigma^2 \sigma^{1*}.$$
Now you can prove the other two identities!
thank you @vanhees71. Yes you are right.
 
  • #6
dear @vanhees71
And 1 more question, do you have any idea to explicitly show that ##\sigma^2 \psi^*_L## transforms like a right-handed spinor? using the identity.
 
  • #7
Take the conjugate complex of the transformation law for ##\psi_L## and multiply with ##\sigma^2##. Then use the (anti-)commutation relation for the complex conjugated Pauli matrices just proven.
 
  • Informative
Likes Pouramat
  • #8
vanhees71 said:
Take the conjugate complex of the transformation law for ##\psi_L## and multiply with ##\sigma^2##. Then use the (anti-)commutation relation for the complex conjugated Pauli matrices just proven.
I have a question to that. Peskin asks us to show that ##\sigma^2\psi_L## transforms like a right-handed spinor. So why can we just consider the transformation of ##\psi_L## alone to then multiply by ##\sigma^2##, and not the transformation of whatever the object ##\sigma^2\psi_L## is?

Edit: I think that my question is why we don't need to take the ##\sigma^2## in front into account when doing the transformation. After all, ##\sigma^2\psi_L\ne\psi_L##, so it should be taken into account when transforming, right?
 
Last edited:
  • #9
You now the transformation representing Lorentz transformations. The infinitesimal version is already sufficient, i.e., the generators for left- and right-handed Weyl spinors. Then it's just Pauli-matrix gymnastics to prove that ##\sigma^2 \psi_L## transforms like ##\psi_R##. Mathematically it's of course not the special pauli matrix ##\sigma^2## which is behind this but the antisymmetric product of two spinors given by ##\epsilon_{ij}## (with ##\epsilon_{12}=-\epsilon_{21}=1##) which is relevant here.

That it's an antisymmetric product instead of a symmetric hints at the fact that one should rather argue with Grassmann-number valued fields for fermions, which is what comes out also using the path-integral formulation.
 
  • #10
vanhees71 said:
You now the transformation representing Lorentz transformations. The infinitesimal version is already sufficient, i.e., the generators for left- and right-handed Weyl spinors. Then it's just Pauli-matrix gymnastics to prove that ##\sigma^2 \psi_L## transforms like ##\psi_R##. Mathematically it's of course not the special pauli matrix ##\sigma^2## which is behind this but the antisymmetric product of two spinors given by ##\epsilon_{ij}## (with ##\epsilon_{12}=-\epsilon_{21}=1##) which is relevant here.

That it's an antisymmetric product instead of a symmetric hints at the fact that one should rather argue with Grassmann-number valued fields for fermions, which is what comes out also using the path-integral formulation.
I’m sorry, but I fail to see how this answers my question. What doesn’t make sense for me is the following:
We are asked to show that the object ##\sigma^2\psi_L^*## (in my previous post I forgot the complex conjugate, sorry) transforms as a right-handed spinor. For me, this means that we need to consider $$\sigma^2\psi_L^*\to(\sigma^2\psi_L^*)’.$$However, you are saying that we should consider $$\psi_L^*\to(\psi_L’)^*,$$to then just multiply by ##\sigma^2##. I can see that this gives the correct result, but I fail to see why we are allowed to do it like this, considering that, in general, these are two different quantities. I hope that this clarifies my confusion.
 
  • #11
But then you solved the problem. I don't understand, what's still unclear.
 
  • #12
vanhees71 said:
But then you solved the problem. I don't understand, what's still unclear.
I wouldn't consider it solved, since I don't know why I am allowed to do the transformation like this. The goal for me is to not blindly solve it and move on, but to understand why it works like this. What I want to know is why I can consider the transformation ##\psi_L^*\to(\psi_L')^*##, even though one is asked to consider the transformation ##\sigma^2\psi_L^*\to(\sigma^2\psi_L)'##, since we are asked to show that ##\sigma^2\psi_L^*## transforms as a right-handed spinor. I fail to see what confuses you in my question...
 
  • #13
Why? If you know how ##\psi_L## transforms, you can just take the conjugate complex of the equation to know how ##\psi_L^*## transforms. Then you multiply by ##\sigma^2## to find out our ##\sigma_2 \psi_L^*## transforms and compare this with the transformation of ##\psi_R##.
 

1. What are Weyl spinors in QFT1?

Weyl spinors are mathematical objects that describe the behavior of spin-1/2 particles in quantum field theory (QFT). They are represented by two-component complex vectors and are used to describe the fundamental building blocks of matter, such as electrons and quarks.

2. What is the significance of the Weyl spinors transformation?

The Weyl spinors transformation is a mathematical operation that transforms one set of Weyl spinors into another set. It is significant in QFT as it allows us to describe the behavior of particles under different symmetries, such as rotations and boosts.

3. How is the Weyl spinors transformation related to the Dirac equation?

The Dirac equation describes the behavior of spin-1/2 particles in QFT and is based on the Weyl spinors transformation. By applying the transformation to the Dirac equation, we can derive equations that describe the behavior of particles under different symmetries.

4. What is the role of Weyl spinors in the Standard Model of particle physics?

In the Standard Model, Weyl spinors are used to describe the behavior of fundamental particles, such as quarks and leptons. They are essential for understanding the symmetries and interactions between these particles, which are the building blocks of matter.

5. How does the Weyl spinors transformation affect the properties of particles?

The Weyl spinors transformation affects the properties of particles by changing their spin, momentum, and energy. It also allows us to describe the behavior of particles under different symmetries, which is crucial for understanding the fundamental forces and interactions in the universe.

Similar threads

  • Advanced Physics Homework Help
Replies
8
Views
700
Replies
3
Views
843
  • Advanced Physics Homework Help
Replies
1
Views
931
  • Advanced Physics Homework Help
Replies
2
Views
3K
  • Advanced Physics Homework Help
Replies
1
Views
1K
  • Advanced Physics Homework Help
Replies
1
Views
1K
  • Advanced Physics Homework Help
Replies
2
Views
430
  • Advanced Physics Homework Help
Replies
3
Views
352
  • Advanced Physics Homework Help
Replies
1
Views
2K
  • Advanced Physics Homework Help
Replies
9
Views
1K
Back
Top